Question

Suppose there are two firms competing in a market. Both firms produce identical products. Firm One...

Suppose there are two firms competing in a market. Both firms produce identical products. Firm One is an efficient firm and has total cost function C1=5q1; Firm Two is a less efficient firm and has total cost function C2=10q2 . Market demand for this product is given by Q=150-2p.

If two firms compete in quantities of production, find out the best response function of each firm and the equilibrium output level of each firm.

0 0
Add a comment Improve this question Transcribed image text
Answer #1


Page No P = 75-0-54 イ4-3. ラ 2 505(44t 65 。。 ot equili biu 91 2 37-560 o 75

Add a comment
Know the answer?
Add Answer to:
Suppose there are two firms competing in a market. Both firms produce identical products. Firm One...
Your Answer:

Post as a guest

Your Name:

What's your source?

Earn Coins

Coins can be redeemed for fabulous gifts.

Not the answer you're looking for? Ask your own homework help question. Our experts will answer your question WITHIN MINUTES for Free.
Similar Homework Help Questions
  • 1. Suppose there are two potential customers in the market. One has demand function D1(p)=10-p ....

    1. Suppose there are two potential customers in the market. One has demand function D1(p)=10-p . The other has demand function D2(p)=20-2p. The only firm in this market has constant marginal cost of 2. (1) Draw the two demand curves in a graph, with price on the vertical axis and demand on the horizontal axis. (2) (3rd-degree price discrimination) If the monopoly can identify the two consumers and charge different prices to them, what is the optimal price charged to...

  • 1. Suppose there are two potential customers in the market. One has demand function D1(p)=10-p ....

    1. Suppose there are two potential customers in the market. One has demand function D1(p)=10-p . The other has demand function D2(p)=20-2p. The only firm in this market has constant marginal cost of 2. (1) Draw the two demand curves in a graph, with price on the vertical axis and demand on the horizontal axis. (2) (3rd-degree price discrimination) If the monopoly can identify the two consumers and charge different prices to them, what is the optimal price charged to...

  • The market for widgets consists of two firms that produce identical products. Competition in the market...

    The market for widgets consists of two firms that produce identical products. Competition in the market is such that each of the firms independently produces a quantity of output, and these quantities are then sold in the market at a price that is determined by the total amount produced by the two firms. Firm 2 is known to have a cost advantage over firm 1. A recent study found that the (inverse) market demand curve faced by the two firms...

  • Suppose there are two firms in a market producing differentiated products. Both firms have MC=0. The...

    Suppose there are two firms in a market producing differentiated products. Both firms have MC=0. The demand for firm 1 and 2’s products are given by: q1(p1,p2) = 5 - 2p1 + p2 q2(p1,p2) = 5 - 2p2 + p1 a. First, suppose that the two firms compete in prices (i.e. Bertrand). Compute and graph each firm’s best response functions. What is the sign of the slope of the firms’ best-response functions? Are prices strategic substitutes or complements? b. Solve...

  • Suppose there are two firms, 1 and 2, competing in quantity. The market demand is p...

    Suppose there are two firms, 1 and 2, competing in quantity. The market demand is p = 15-(q1 +q2), where q1 and q2 are the quantities produced by rms 1 and 2. Both rms have constant marginal cost c1 = c2 = 3. (a) [10] Find the Cournot equilibrium of this market. Compute the consumer surplus in equilibrium. b) Now suppose firms 1 and 2 merge, so that they become a monopolist with demand function p = 15 ? q,...

  • Consider two firms competing in a market with a demand function P=150-Q. Both firms have constant...

    Consider two firms competing in a market with a demand function P=150-Q. Both firms have constant marginal cost c>0. There are no fixed costs. They compete by setting prices p₁ and p₂ simultaneously. (Bertrand game.) Which of the following statements is not correct? Select one: a. Both firms charging charging p = c is a Nash equilibrium. b. When firm 1 sets  where  is the industry monopoly price, firm 2's best response is to set . c. When p₁=c, any price p₂≥c...

  • Two firms sell identical products and compete as Cournot (price-setting) competitors in a market with a...

    Two firms sell identical products and compete as Cournot (price-setting) competitors in a market with a demand of p = 150 - Q. Each firm has a constant marginal and average cost of $3 per unit of output. Find the quantity each firm will produce and the price in equilibrium.

  • Two firms are participating in a Stackelberg duopoly. The demand function in the market is given...

    Two firms are participating in a Stackelberg duopoly. The demand function in the market is given by Q = 2000 − 2P. Firm 1’s total cost is given by C1(q1) = (q1) 2 and Firm 2’s total cost is given by C2(q2) = 100q2. Firm 1 is the leader and Firm 2 is the follower. (1) Write down the inverse demand function and the maximization problem for Firm 1 given that Firm 2 is expected to produce R2(q1). (2) Compute...

  • Suppose a market has two firms that sell identical products. These firms face an inverse market...

    Suppose a market has two firms that sell identical products. These firms face an inverse market demand function of P=120 – Q. Firm 1 has a constant MC=20. Firm 2’s marginal cost is MC=30. Find the Cournot equilibrium price, quantities, and profits for each firm. If these firms were able to perfectly collude, what would be the monopoly equilibrium?

  • Consider two firms (Firm A and Firm B) competing in this market. They simultaneously decide on...

    Consider two firms (Firm A and Firm B) competing in this market. They simultaneously decide on the price of the product in a typical Bertrand fashion while producing an identical product. Both firms face the same cost function: C(qA) = 12qA and C(qB) = 12qB, where qA is the output of Firm A and qB is the output of Firm B. The demand curve is P = 30 - Q. (i) What will be the Bertrand-Nash equilibrium price (pB) chosen...

ADVERTISEMENT
Free Homework Help App
Download From Google Play
Scan Your Homework
to Get Instant Free Answers
Need Online Homework Help?
Ask a Question
Get Answers For Free
Most questions answered within 3 hours.
ADVERTISEMENT
ADVERTISEMENT
ADVERTISEMENT